LSAT and Law School Admissions Forum

Get expert LSAT preparation and law school admissions advice from PowerScore Test Preparation.

 Administrator
PowerScore Staff
  • PowerScore Staff
  • Posts: 8916
  • Joined: Feb 02, 2011
|
#26417
Complete Question Explanation
(The complete setup for this game can be found here: lsat/viewtopic.php?t=7428)

The correct answer choice is (B)

If F and H are assigned to the same story, then they must be in different fields. This results in the writer’s assistants being G, L, and F, and the photographer’s assistants being K, H, and J. The photographer’s assistant row is the most restricted because K cannot be assigned to S, and must instead be assigned to R. Thus, H must be assigned to S. Accordingly, F is assigned to H in the writer’s row, with G and L rotating between R and T:
powerscore_M12_T3_J2010_LG_explanations_game_4_#20_diagram_1.png
G and L are the only uncertainties in this Could Be True question, and you should focus on answers that contain them. This narrows the consideration to answer choices (B) and (E). Answer choice (B) properly reflects the fact that G can be assigned to R, and thus answer choice (B) is correct.
You do not have the required permissions to view the files attached to this post.

Get the most out of your LSAT Prep Plus subscription.

Analyze and track your performance with our Testing and Analytics Package.